Transformers and effect of eddy currents

Поділитися
Вставка
  • Опубліковано 2 кві 2012
  • This deals with a question about the electromagnetism video and concerns the effect of eddy currents in the core of a transformer
  • Наука та технологія

КОМЕНТАРІ • 71

  • @anthonythomas2281
    @anthonythomas2281 9 років тому +10

    DrPhysicsA, cheers for helping a fellow brotha out....to all my fellow a level companions....we will get through this...

    • @yoyoyoc3po
      @yoyoyoc3po 6 років тому

      Anthony Thomas I need a crap TV to watch teletubbies

  • @DrPhysicsA
    @DrPhysicsA  12 років тому

    Latest video on EPR paradox (simple explanation) now uploaded. Will do Schrodinger's cat next.

  • @sandlertossone1813
    @sandlertossone1813 4 роки тому

    The breaking effect is the coolest thing ever. Great video

  • @robertmattison1282
    @robertmattison1282 6 років тому +1

    I wish he was my teacher when I was in school. Thanks for posting this on UA-cam

  • @mahyar7126
    @mahyar7126 5 років тому

    Thanks alot,,Ive seen many videoes and got nothing ,,but your explanation is amazing.

  • @DrPhysicsA
    @DrPhysicsA  10 років тому +1

    They are usually surface currents which arise whenever the electrons on that surface are subject to some kind of electric field (eg in EMR).

  • @LithicStudios
    @LithicStudios 9 років тому

    This is a great explanation

  • @tanmoydutta5846
    @tanmoydutta5846 6 років тому +4

    Sir, the direction of the current at 6:23 is wrong... because from the lower surface, according to the clock rule, we must have an anticlockwise direction of current in order to produce the north polarity so as to oppose the north polarity of the magnet being moved towards it.....

  • @DrPhysicsA
    @DrPhysicsA  11 років тому

    Thanks. Yes you are right. I thought I had added an annotation but I will do so now. Thanks again.

  • @Stewartthorp
    @Stewartthorp 8 років тому

    great explanation. thanks

  • @rongroskey5177
    @rongroskey5177 10 років тому +2

    On the DC coil the electron flow is always from the negative to the positive, the same as a battery. I really would like to see the conventional explanation of current flow buried along with Franklin. Convention of current flow teaches confusion.

  • @markben145
    @markben145 5 років тому

    Although not all the explanation flows and professional good to hear this teacher teaches

  • @annyan904
    @annyan904 5 років тому

    love your videos and green sketchpen

  • @enzoestayscabini2487
    @enzoestayscabini2487 7 років тому +2

    I am building an electric motor and i dont understand why when I put
    close to mine electro magnet any ferromagnetic material or a magnet , it
    start to vibrate?
    By the way does the transformer vibrate becouse the eddy current?
    thank you in advance

  • @DrPhysicsA
    @DrPhysicsA  10 років тому

    It is an induced current and will therefore flow in such a way as to create a magnetic field which opposes the field which created it in the first place.

  • @mrabal3
    @mrabal3 12 років тому

    As the disk approaches the field, the 'change' in field value is up (the field is up and getting bigger in time). In this case, the induced current at the leading edge will be directed (not flow, for charge is what flows) in a direction out of the screen, perpendicular to the direction of motion, so that if you were observing the disk from above, the current would be directed in a clockwise direction; this is a very tricky concept.

  • @DrPhysicsA
    @DrPhysicsA  11 років тому

    Any experiment which has a magnetic field and a wire with one of them moving such that there is a rate of change of flux and hence a current flowing in the wire will suffice. That current will itself generate a magnetic field. Lenz's law says that the magnetic field so generated will be such as to oppose the field which caused the current to flow.

  • @TSulemanW
    @TSulemanW 5 років тому

    nicely explaination

  • @leviterande
    @leviterande 11 років тому

    Thanks! yes but I was wondering about a varying Ac magnetic field? would it still be attracted? isn't there supposedly eddy currents in the iron that oppose the varying magnetic field? Thanks

  • @ojtechml
    @ojtechml 9 років тому

    Regarding the lamination of the core, does it affect the permeance but doesn't affect the permeability because that's inherent to the said material ie constant ?

  • @QuinnWaters
    @QuinnWaters 6 років тому

    thank you!

  • @ayushmishra24
    @ayushmishra24 7 років тому

    awesome😊

  • @RidhoIbrahim
    @RidhoIbrahim 8 років тому

    Thank you :)

  • @dsrmanav5560
    @dsrmanav5560 4 роки тому +6

    6:15, the current must be in clockwise direction

    • @yoyo18200
      @yoyo18200 3 роки тому

      how do we know

    • @Nicknarw
      @Nicknarw 3 роки тому

      I made the same conclusion! Been reading back and studying past chapters in my books trying to make sense of why he said it was counter clockwise!

    • @Flqmmable
      @Flqmmable 2 роки тому

      correct

  • @Ghostpalace
    @Ghostpalace 10 років тому

    With alternating signals, you have to include a cos(phi) into the power equation.

  • @bauyrzhankokanov5498
    @bauyrzhankokanov5498 4 роки тому

    Thank you

  • @DrPhysicsA
    @DrPhysicsA  11 років тому

    Just consider holding a magnet above a piece of iron. The iron would be attracted to the magnet.

  • @susimadesilva7185
    @susimadesilva7185 9 років тому +1

    tnx DrPhysicsA

  • @mrabal3
    @mrabal3 12 років тому

    It might help to recall the Lorentz Force Law here. This states that a force will arise that is perpendicular to the direction of the product of the velocity vector and the magnetic field vector as dictated by the right-hand rule, which, if you apply it here, will be off to the left. That is what stops the pendulum.

  • @Greg_Chase
    @Greg_Chase Рік тому

    A challenge for students new to transformers is the constraint on maximum achieve-able current in the secondary circuit of a step-up transformer.
    Since the power in the primary circuit, Pp = VpIp, must equal the power in the secondary's circuit, Ps = VsIs, we have the following situation for a 1:10 step up transformer with 10 volts, 1 amp max in the primary:
    1) the max power in the primary winding's circuit is: (10 volts) x (1 amp) = 10 watts
    2) voltage in the secondary winding is stepped-up by a factor of 10, so the secondary voltage Vs = (10 volts from the primary) x (stepped-up 10 times) = 100 volts
    3) Since the power Ps in the secondary MUST be 10 watts, we have: 10 watts = 100 volts x Is, so that current in the secondary winding circuit "Is" must be: (10 watts) / (100 volts) = 0.1 amps
    Fine. We have (0.1 amps) x (100 volts) = 10 watts in the secondary winding.
    HERE'S WHAT CONFUSES PEOPLE new to transformers:
    By limiting the maximum current possible in the secondary winding, the 100 volts in the secondary seems "compromised", or "anti-physical" because, if you present a circuit with a 100 ohm resistor and say "the voltage in the circuit is 100 volts - what is the max current in the circuit with the 100 ohm resistor?"
    They are NOT going to say "it would be 0.1 amps". They are going to say "it will be (100 volts) / (100 ohms) = 1 amp"
    When you then tell them "No, this circuit is special. It is a circuit of the secondary winding of a transformer, and the maximum current can only be 1/10th of that - so, not 1 amp, only 0.1 amps"
    When you present your case as to why the 100 volts in the secondary circuit is 'compromised' and cannot reach 1 amp of current, it can undermine their faith in your explanations.
    They feel like you're trying to tell them "sometimes, 100 volts gets you 1 amp. Then, other times, only 100 milliamps. And still other times, say - for a 1:100 step up transformer, your 100 volts in the secondary only gets you TEN MILLIAMPS."
    CRUX: when a 'reactive' component - an inductor and/or capacitor - is in the circuit, Ohm's law will not suffice. (The secondary winding of the transformer is obviously an inductor, and to figure out current, you rely on the impedance, Z, calculation. The inductive reactance Xc is: 2piFL, where F = frequency, L = inductance.
    .

  • @voodoo2337
    @voodoo2337 11 років тому

    yes so you saying that inducted electricity travels easier trough iron instead of air that is clear but I was talking about isolating the wiring somehow from the eddy current so it won't jump over the wiring and make short circuit

  • @DavidutzAlmy
    @DavidutzAlmy 9 років тому +1

    If i am not mistaken the magnetic breaking is achieved due to the electromagnetic force ( dF=i dl x B ) What i would like to ask is if the currents are circular, wouldnt a line integral of the force be zero resulting in no damping?

    • @ufo717212
      @ufo717212 8 років тому

      +Ben Dover The circular current is not completely within the room between N and S, where B is effective The actual braking happens when the metal plate is entering/exiting that room. During that time, the Ampere force is only applied to one part of the circuit while the other part is outside the magnetic field . Therefore the line integral of force can only be performed on the affected part of the circuit not the whole closed circuit. So F is non-zero then.
      Check this image of Eddy current braking from WIkipedia: en.wikipedia.org/wiki/Eddy_current#/media/File:Eddy_current_brake_diagram.svg
      The metal plate is usually much larger than the area of the magnetic filed, so it is entering/exiting the room between N and S all the time resulting in continuous braking force . Eddy current forms surrounding the two edges of the magnetic filed. Currents between N/S are consistent in direction(Lorentz Law) (so are the Ampere forces), while the rest of circular currents are outside the magnetic filed.

    • @DavidutzAlmy
      @DavidutzAlmy 8 років тому

      Thank you very much!ukyee

  • @user-eu8ro3lx7g
    @user-eu8ro3lx7g 8 років тому

    great explanation!. but i think the laminated core direction is something wrong in eddey current part.

  • @therealjordiano
    @therealjordiano 11 років тому

    if you put the coils in a vacuum then the magnetic field produced by one of the coils will weaken the further away you get.. so you need to put some iron in there (because iron has a high permeability) so that the magnetic field strength doesn't weaken very much

  • @universalsailor
    @universalsailor 10 років тому +1

    At 16.45 the drawing of the laminated core is not right, surely. The laminations should be such that each one forms a complete rectangle, thus constraining the eddy currents to 2-D so to speak.

  • @voodoo2337
    @voodoo2337 11 років тому

    I know what is the EPR paradox and its answer: waves can't be emitted at perfect strait trajectory from the emitter because its like the shotgun shells when you shoot the shell the buckshot is traveling trough the pipe(in light's case - fiber optic) at strait line but when it leaves the cylinder it spreads wider with distance same thing with the waves only one particle in the bulb is emitting only one light wave, thing about it if light comes from another galaxy you might not see it if...

  • @ollywild925
    @ollywild925 10 років тому

    The definition off the 3-D wires can be unclear., I don't know which bit of the wire is c

  • @Bishen007
    @Bishen007 11 років тому

    Sir can u pls expain the outline the experiment of Lenz's Law.. Thanks..

  • @naveed3621
    @naveed3621 10 років тому

    Magnetic Field in a solenoid is equal to (nuI) where n=N/l

  • @MacTwsh
    @MacTwsh 10 років тому

    in 13:45 i didnt understand why the eddy current creates magnetic field down?? according to right hand rule it must be up

  • @jeanpierredaviau7478
    @jeanpierredaviau7478 7 років тому

    Hi, I am looking for a formula to calculate the length of a rope enrolled flat on a deck. Kind of spiral enrollment every boat lover have seen.

  • @houstonxue9233
    @houstonxue9233 8 років тому +5

    isn't the current flow direction at 6:15 in the loop wrong?

    • @Azagro
      @Azagro 8 років тому

      +Houston Xue Right hand rule.

    • @carreraGT92
      @carreraGT92 8 років тому +6

      +Houston Xue Yes, there would be magnetic field lines going up out of the page inside the loop of wire thus the North side of the magnet would actually be attracted to that. So the direction of current flow should be the other way to 'push' the magnet down

  • @leviterande
    @leviterande 11 років тому

    Ima having a hard time understand one small detail. when a varying magnetic field exists above a ferromagnetic material, would there be attraction? repulsion? or basically ZERO?

    • @marcisberzins
      @marcisberzins 4 роки тому

      In general, there will be an attraction, because ferromagnetics attracts to a magnet regardless of the pole.

  • @skyview1997
    @skyview1997 10 років тому

    When are eddy currents produced?

  • @shubhammali1634
    @shubhammali1634 6 років тому

    Plz explain derivation of efficiency of transformer..

  • @joedarkmatter5051
    @joedarkmatter5051 5 років тому

    I'm the 56,000-th viewer! :)

  • @ugi991
    @ugi991 8 років тому

    Can someone please tell me when to use Fleming's left hand rule and when to use the right hand rule? I keep confusing them! if someone has a method where I can memorise those please help

    • @ericb4652
      @ericb4652 8 років тому

      Use the left-hand rule when you're dealing with current-carrying conductors in a magnetic field. Use the right-hand rule when you're dealing with electromagnetic induction. I don't have a fun or interesting way to remember that though, sorry :)

    • @DrPhysicsA
      @DrPhysicsA  8 років тому +3

      In the UK we know that motor cars drive on the left hand side of the road. So LH rule for MOTORs and RH rule for generators.

    • @ayushmishra24
      @ayushmishra24 7 років тому

      yeah sure...
      to find induced current use right and to find direction of magnetic field due to flowing current use left..

  • @blusky3591
    @blusky3591 9 років тому

    As per the Ohm's Law, Voltage is Directly Proportional to Current. So when current is transformed in a set up transformer why is it like, there is high voltage but less current flowing. It shouldn't be that way though because as voltage increases Current should increase too. So why does that happen. Please give me the technical reason as well as the Maths one as I am curious to know how does that happen ... Thanks inadvance.

    • @DrPhysicsA
      @DrPhysicsA  9 років тому +1

      The key point here is that power is the same in theory (leaving aside heat losses etc). Since power = VI then if one increases the other must decrease.

    • @blusky3591
      @blusky3591 9 років тому

      DrPhysicsA Thanx sir but i was just wondering that how can low current floe even when high voltage is applied across.

    • @The112Windows
      @The112Windows 8 років тому

      +Ganesh P IV= constant Therefore is V increases I must decrease.

  • @georgerelax7302
    @georgerelax7302 10 років тому

    if you put a voltometer there you will see 0 volts..close the circuit or your coil has no voltage..

  • @arainbros1471
    @arainbros1471 8 років тому

    why can 't we find the power loses by v^2/R ?

    • @DrPhysicsA
      @DrPhysicsA  8 років тому +1

      +Arain bro's You can. But the v in the equation must be the potential drop across the transmission line not the full voltage across the whole system.

    • @arainbros1471
      @arainbros1471 8 років тому

      got it, and by the why is it possible to obtain multiple outputs ? like if we just introduce two secondary windings instead of one?

  • @voodoo2337
    @voodoo2337 11 років тому

    ... the waves don't "meet" your telescope so that is one of the reasons the stars on the night sky are blinking ::)

  • @voodoo2337
    @voodoo2337 11 років тому

    JUST PUT THEM IN VACUUM!!!

  • @mrabal3
    @mrabal3 12 років тому

    I don't think you have it right...you are stating Lenz's law incorrectly...you are leaving out the all-important word 'change', i.e., the induced current in the disk of the pendulum will flow in a direction which creates a magnetic field that is in a direction that opposes the 'change' in the magnetic field that induced it (not the field itself; these can be totally opposite).